Can someone explain this necessary assumption problem to me? I got the question wrong the first time and during blind review (I answered B, then E). The answer doesn't seem to be A or C, so it must be D. But why?
For example, in PT 93, JY explains that the ... other in/out games. In PT 61, I tried applying the ... a conditional relationship similar to PT 93 is only when there ...
I'm trying to identify flaws. is PT 56 S3 Q10 an equivocation flaw; can you use more than a word but a concept in this type of flaw?
Is PT 52 S3 Q4 a false appeal flaw?
Is PT 54 S4 Q16 an implication flaw ?
thanks
I figured since there is no explanation video on this one, I'd offer my two cents. Please feel free to respond if my explanation is lacking anything or if you have a better one.
So the correct AC is B and here is why. The stimulus tells us ...
This is a great post! Thank you for the positive vibes! Wishing us all peace of mind for the next few weeks! In solidarity with those of us battling PT79! Crush. This. Test.
... always funny to look at PT79 from the big picture intellectually ... " (because when I bomb a PT I just learn from it ... ). May all my fellow 7sage PT79 takers win that battle!
... 're going to see is PT79. While every test is valuable ... be expecting on game day. 79 is a great example actually ... a new game type in 79. When do you want to ...
@CrushLSAT said:
question 19 (which asks for a complete and accurate list of doctors at Souderton) be E (N and P)? Why can't the ~N --> J pair be treated the same as ~O --> J? If we only have N and P, aren't we still good since we have at ...